« first day (3760 days earlier)      last day (1254 days later) » 

9:04 PM
I haven't done that in a long time, but cant you do it saying it is either prod of polynomials of degree 3, 3 or 2, 4
And then bla bla $\omega$ times roots bla bla must be 3,3
And then do direct computation
 
Some more fast-spoken nonsense: x^3 - sqrt(3) and x^3 + sqrt(3) are both irreducible over Q(sqrt(-3), i) = Q(sqrt(3), i) because they are irreducible over Q(sqrt(3)) by Eisenstein and if it factored in Q(sqrt(3), i) they must have a root in there as well, impossible because Q(sqrt(3), i)/Q is a degree 4 extension whilst Q[x]/(x^6 - 3) is degree 6. (@Thorgott check?)
 
I also don't see why Eisenstein's doesn't work directly
 
How?
 
Do i misremember it? It is basically pick a prime that divides all coefficients except the first one right
Also $p^2$ shouldnt
 
but this is irreducibility over Q(sqrt(-3))
not Q
 
9:09 PM
Oh right wtf yeah
How did I miss that
 
Oh btw why I am doing this? x^3 - sqrt(3) is irreducible over Q(sqrt(-3), i) because if it factored a root would be in Q(sqrt(-3), i), which is a degree 4 extension of Q, but x^6 - 3 is a degree 6 poly
 
wait, isn't just $x^6-3=(x^3-\sqrt{3})(x^3+\sqrt{3})$
 
Q(sqrt(-3)) man
 
@love_sodam Let $K=\Bbb Q(\sqrt[6]{3})$ and $L=\Bbb Q(\sqrt{-3})$. If $x^6-3$ is not irreducible over $L$, then $[KL:L]<6$. Using the tower law we have $[KL:K][K:\Bbb Q]=[KL:\Bbb Q]=[KL:L][L:\Bbb Q]<12$ Now pluggin in $[K:\Bbb Q]=6$ gives $[KL:K]<2$, so $[KL:K]=1$ which implies $L \subset K$. But this is nonsense: $K$ is real and $L$ not
 
@BalarkaSen don't get what you do here
 
9:12 PM
Forget it
Screw this
 
@LukasHeger this
 
@Krijn ?
 
I was mentioning that what you said makes a lot of sense, as a signal to bala
 
ah I see
 
I already gave a proof, I just couldn't be arsed to write it properly. x^3 +- sqrt(3) is irreducible over Q(sqrt(-3), i), hence x^6 - 3 is irreducible over Q(sqrt(-3))
 
9:15 PM
Seems @LukasHeger make sense
 
@TedShifrin please flag those. I hate that. I will undelete those immediately.
 
@BalarkaSen yeah that works as well. Generally if $L/K$ is Galois with Galois group $\Gamma$ and $f \in L[x]$ is irreducible, then $\prod_{g \in \{\gamma f\mid \gamma \in \Gamma\}} g$ is irreducible over $K$
 
yeah
just fucking annoying to write, i dont give a shit
field theory is garbage
 
:(
I feel u undervalue the elegance of some field theory
 
I have learnt and forgotten it 80 times already
I don't undervalue it it just annoys the hell out of me
 
9:23 PM
Ever learnt about complex multiplication?
 
i know how to multiply in C yes
 
"The theory of complex multiplication is not only the most beautiful part of mathematics but also of the whole of science."
- Hilbert
That's a reason to study it, no?
 
"RP^2 does not immerse in R^3"
-also Hilbert
Reason enough to not trust this man
 
"can't solve these 23 problems, pls fix" -Hilbert
Solve your own problems old man
 
"We are a university, not a bathhouse"- Hilbert
 
9:26 PM
yeah Poincare didn't make a list of problems he just conjectured 1 thing
fuckin Hilbert
 
I have a Hilbert
I mean a question.
what is a power set map?
 
That's why I like Weil
Make sensible conjectures, guide a field, in the end there is a nice theory that works
And you only need a Grothendieck and Deligne to solve it
 
Is there algebraically closed field contained in C?
 
"only" Grothendieck and Deligne
 
which is not C
 
9:29 PM
Qbar
 
@love_sodam Yes, plenty
 
many embeddings of Qbar :p
 
what is Q bar?
 
"look at my fancy hat" - Hilbert
4
 
9:30 PM
oh algebraic closure you mean
 
too much Hilbert in the chat man
its triggering me
new topic
 
@BalarkaSen Plenty of embeddings of C as well
 
lol
ok dude
 
But I want explicit example not like closure something
I heard that
 
"all algebraic numbers" is not an explicit example?
what does explicit mean
what does example mean
 
9:31 PM
Jokes apart there's also embeddings of $\overline{\Bbb Q(x)}$ and so on
 
You'll need the bar right
Every algebraically closed field thats not Q bar seems more complex then Q bar
 
all algebraic numbers is the easiest example
it's the smallest one contained in C
 
Let A be the field of all algebraic numbers
@AlessandroCodenotti "Jokes apart"
 
I heard that as an example of infinite algebraic extension, {a\in C: a is algebraic over Q}
Is this algebraiclly closed?
 
9:33 PM
Lol
 
yes
 
Q-bar
 
that's the algebraic closure of Q
 
Oh that's Q bar?
 
ye
 
9:33 PM
Oh I didn't know that
 
Q-bar sounds like a bar
 
its a good exercise to show if a, b are algebraic over Q then so are a + b and ab
 
I don't know closure so can you prove it directly from that?
 
yes you can
 
it's actually quite tedious isn't it
 
9:34 PM
I think so
 
which is?
 
ab in particular. you get the polynomial from some stupid determinant
 
no, that's easy
 
finite extension iff algebraic
 
^
 
9:35 PM
Q(ab)/Q(a)/Q
tower law
 
what's the polynomial
 
not iff
 
but showing that the algebraic numbers are algebraically closed is tedious
 
@MikeMiller nobody cares
you dont need to know it
 
geometry brain
 
9:35 PM
hahah
 
@BalarkaSen thx
 
@Thorgott is this what I'm thinking of?
there's a determinant somewhere
 
a,b is algebraic then a+b, ab is algebriac is easy but showing that collection is algebraic ally closed seems not that easy
 
perhaps Mike
 
@Thorgott that follows from transitivity of algebraicness if $E/L/K$ is a tower and $E/L$ is algebraic and $L/K$ is algebraic, then $E/K$ is algebraic
 
9:36 PM
how?
yeah
what Lukas said
What are you people smoking
 
this is stupid
 
so if $E/\overline{\Bbb Q}$ is algebraic, then actually $E\subset \overline{\Bbb Q}$
 
Give me some of this
 
You can't tell me a number is algebraic and not be able to write down a polynomial
 
But transitivity of algebraicness is not trivial right
 
9:37 PM
its tower law!!
 
yeah polynomials
 
"this is algebraic by the theory of nilpotent schemes"
 
finite iff algebraic
 
thats false balarka
 
it's just tower law as Balarka says
 
9:37 PM
ok finite implies algebraic
 
yeah
 
Q bar is an example
 
you can always reduce questions about algebraicness to questions about finite extensions
 
yeah
direct limits
 
algebraic iff colimit of finite extensions
 
9:38 PM
everything is a direct limit offinite extensions
COLIMIT FUCK OFF
 
If you can't turn this into an algorithm to find the polynomial defining x^3 + sqrt(2)x + 1 = 0 what's the point
 
just say union
 
Balarka seems aggresive tonight
 
I am getting triggered by (1) field theory (2) Hilbert
get this shite out of my face
 
Still $\Rightarrow$ CM
 
9:39 PM
@BalarkaSen Just take a filtered colimit bro
 
@BalarkaSen let's talk about Hilbertian fields
 
Hilbert class field lmao
Lets talk about maximal abelian covers instead
how do you build those
and what does knot polynomials have to do with them
 
you can also prove it using determinants, Mike, but why would you
 
Please someone steer this away from Field Theory
 
I veto your idea balarka
 
9:40 PM
@Krijn He's dielectric.
 
Also this ^
 
@AlessandroCodenotti can one use löwenheim-skolem here
 
@BalarkaSen Lmao you've been struggling with electric field theory all day and now it's algebraic field theory instead
 
when i SAY divert this AWAY from field theory I DONT MEAN SET THEORY
 
And now you want to move to topological quantum field theory?
 
9:41 PM
haha
 
So confusing
 
DONT DO MODEL THEORY NOW
 
@BalarkaSen can't stop me
 
@MikeMiller Lmfao
 
I can embed the complex numbers into super duper nonstandard complex numbers of whatnot cardinality and shits
 
9:42 PM
I'm laughing irl
 
Clearly we need to move to class field theory
 
Tell me something interesting that's a consequence of model theory. The only example I know is Ax-Grothendieck
 
what was that Tarski garbage
 
@user2103480 Hm not sure
 
existential quantifiers can be replaced by universal ones in algebraic formulas
 
9:43 PM
@MikeMiller Tarski-Seidenberg
 
yeah that
 
@MikeMiller solvability of euclidean geometry
 
o-minimal structures
Tame Topology
 
In relation to what balarka said
 
nerdoid shit
 
9:44 PM
This is the least interesting theorem I've ever read
I've been more excited by technical lemmas
 
LOL
 
@user2103480 Uninteresting unless you have an algorithm
 
@MikeMiller We have an algorithm
its running time is just ungodly
 
make it do all of Hall & Stevens or whatever
solve Euclidean geometry
 
@MikeMiller It's actually a natural example of a doubly exponential algorithm
 
9:46 PM
We also have an algorithm to prove every true assertion coming from ZFC in finite time, believe it or not
 
And the complexity is optimal
 
It's literally just listing consequences of the axioms though kek
 
Lol
 
There's an expspace algorithm
 
stupidity manifest
 
9:47 PM
logic brain
 
@user2103480 That's a semi algorithm or whatever
You can't tell me if something is false in finite time
 
LOL
how do people take this stuff seriously
 
@AlessandroCodenotti algebraically closed field with characteristic zero should be first-order axiomatizable, but not sure how to handle transcendence base
 
@user2103480 Sure, it's even a model complete theory with QE, but I still don't see how to get embeddings out of LS
 
@BalarkaSen Meh it's their game just like we have ours
 
9:49 PM
@MikeMiller if it's false, its converse will show up as well
negation
since that is true
 
independence intensifies
 
If we know it's true, it will be proven in finite time. But we cannot in finite time decide whether it will be proven, generally
 
@MikeMiller agree but it is my individual right to call some games stupid
for example Dungeons & Dragons
Call of Cthulhu pen and paper RPG
 
@user2103480 that's useless right if you don't know if is independent from ZFC
 
exactly
@BalarkaSen like banach-mazur games
looking at the general topology guy in this round
 
9:52 PM
Markov winning strategies
 
@TedShifrin $\frac\pi4$. I didn't see another answer, but I could have missed it.
 
@AlessandroCodenotti ah we can just take ultraproducts instead of löwenheim-skolem (which is about the same, but ensures the embedding of C into the field)
something something cardinality then shows that it's not actually C, hopefully
 
@BalarkaSen well, I guess I won't be inviting you to our weekly get together, when we can have them again.
 
haha
 
@user2103480 Uncountable algebraically closed field of fixed characteristic are classified by cardinality
 
10:02 PM
@robjohn This was for geocalc to see an application of integrals being independent of a parameter, but I don’t think he thought about it even one second.
And I did flag that question, robjohn. Don't know if any mod dealt with it.
 
@AlessandroCodenotti I read it backwards lol I thought it was about fields that contain C, sorry bout that
 
@TedShifrin Yeah, but I don't know if he would have seen why it was independent of $n$, though the proof is simple, just not necessarily obvious.
 
I see what should be $n$ but not why it's independent of $n$
 
@BalarkaSen Substitute $x\mapsto\frac1x$, then add the two.
 
Oh, insane
Nice
 
10:09 PM
the part that is hard is that often one goes down a rat hole looking at complex analytic approaches.
I did for a few minutes after I saw the question.
 
I assigned it to my class for differentiation under the integral.
 
@TedShifrin Ouch! another rat hole. I was thinking of looking at that, but tried $x\mapsto\frac1x$ first, thank heavens.
 
Nah, it works fine.
 
@TedShifrin really? I'll have to look at that.
 
So I recollect.
 
10:16 PM
@TedShifrin Okay, you use the same substitution to show that the derivative is $0$.
 
I think I used $\pi/2-x$
Need to think again. It's been zillions of years.
 
@TedShifrin Ah, yes... you are using $\tan(\theta)$. I had substituted $\tan(\theta)=x$
Then $x\mapsto\frac1x$ is the same as $\theta\mapsto\frac\pi2-\theta$
 
Right. The log suggests it strongly.
 
$$\int_0^{\pi/2}\frac{\mathrm{d}\theta}{1+\tan^n(\theta)} =\int_0^{\pi/2}\frac{\tan^n(\theta)\,\mathrm{d}\theta}{1+\tan^n(\theta)}$$ upon substituting $\theta\mapsto\frac\pi2-\theta$. Average the two.
 
Question: If $T$ is an operator on a Hilbert space $\mathcal{H}$ with the property that, for any bounded sequence $(x_n)$ in $\mathcal{H}$ the sequence $(Tx_n)$ has a convergent subsequence, does this imply that $\text{Im}(T)$ is complete? I feel like the answer is "yes, clearly," but I'm in a headspace of doubting myself at the moment.
 
10:27 PM
@Rithaniel compact you mean
 
$$\int_0^\infty\frac{\mathrm{d}x}{\left(1+x^2\right)\left(1+x^n\right)}=\int_0^\infty\frac{x^n\,\mathrm{d}x}{\left(1+x^2\right)\left(1+x^n\right)}$$ upon substituting $x\mapsto\frac1x$. Average the two.
 
Yeah, the overall goal is to show that $T$ is compact, but this completeness thing is a pit-stop I'm making in my proof.
 
Then you're done pretty quickly
 
The idea is "complete $\to$ orthonormal basis $\to$ finite dimensional $\to$ compact"
Is this too much of a convoluted path, maybe?
 
For compact operators you consider the closure of the image
So you consider the closure of the image of the unit closed ball, in which every bounded sequence is the image of a bounded sequence in the domain, yada yada
 
10:49 PM
Hello all, I have a question about accuracy of converting decimal to binary floating-point number, and to decimal. Please help if possible, thank you.

https://math.stackexchange.com/questions/3914836/proof-of-accuracy-of-converting-decimal-to-binary-floating-point-number-to-decim
 
11:41 PM
@user2103480 So, my attempted proof failed because I falsely thought that complete implied separable. However, your proof also doesn't make any sense to me. I'm trying to go from "all sequences in the image of $T$ have convergent subsequences" to "if $x_n$ is a weakly convergent sequence, then $Tx_n$ is a strongly convergent sequence."
 

« first day (3760 days earlier)      last day (1254 days later) »